www.vorhilfe.de
- Förderverein -
Der Förderverein.

Gemeinnütziger Verein zur Finanzierung des Projekts Vorhilfe.de.
Hallo Gast!einloggen | registrieren ]
Startseite · Mitglieder · Impressum
Forenbaum
^ Forenbaum
Status VH e.V.
  Status Vereinsforum

Gezeigt werden alle Foren bis zur Tiefe 2

Navigation
 Startseite...
 Suchen
 Impressum
Das Projekt
Server und Internetanbindung werden durch Spenden finanziert.
Organisiert wird das Projekt von unserem Koordinatorenteam.
Hunderte Mitglieder helfen ehrenamtlich in unseren moderierten Foren.
Anbieter der Seite ist der gemeinnützige Verein "Vorhilfe.de e.V.".
Partnerseiten
Weitere Fächer:

Open Source FunktionenplotterFunkyPlot: Kostenloser und quelloffener Funktionenplotter für Linux und andere Betriebssysteme
Forum "mathematische Statistik" - Konvergenzgeschwindigeit
Konvergenzgeschwindigeit < math. Statistik < Stochastik < Hochschule < Mathe < Vorhilfe
Ansicht: [ geschachtelt ] | ^ Forum "mathematische Statistik"  | ^^ Alle Foren  | ^ Forenbaum  | Materialien

Konvergenzgeschwindigeit: Frage (überfällig)
Status: (Frage) überfällig Status 
Datum: 13:51 Do 19.01.2012
Autor: blascowitz

Aufgabe
Es ist zu zeigen:

[mm] $E\left[ \left( \frac{1}{n}\sum\limits_{j=1}^{n}\cos(t(X_{j}))-\mu\right)^{2k} \right]=\mathcal{O}(n^{-k})$ [/mm] für [mm] $k\in \IN$ [/mm]


Dabei ist [mm] $\mu \in [/mm] (0,1] $ der Erwartungswert von [mm] $\frac{1}{n}\sum\limits_{j=1}^{n}\cos(t(X_{j}))$ [/mm] und die [mm] $X_{j}$ [/mm] sind u.i.v.,besitzen eine Lebesgue-Dichte und $t [mm] \in \IR$ [/mm] ist fest.




Hallo,

im Zuge meiner Diplomarbeit hab ich in einem Paper diese Behauptung in noch allgemeinerer Form gefunden. Zwecks Übersichtlichkeit habe ich mir erstmal einen Spezialfall rausgenommen.

Meine Beweisidee wäre jetzt vollständige Induktion. Für $k=1$ steht dort einfach die Varianz der Zufallsvariable [mm] $\frac{1}{n}\sum\limits_{j=1}^{n}\cos(t(X_{j}))$. [/mm] Dann ergibt sich die Behauptung für $k=1$ einfach mit den Verschiebungssatz und  [mm] $|cos(t)|\leq [/mm] 1$.

IV: [mm] $E\left[\summe\limits_{l=0}^{2k}\vektor{2k \\ l}\left(\frac{1}{n}\sum\limits_{j=1}^{n}\cos(tX_{j})\right)^{2k-l}\cdot (-\mu)^{l}\right]=\mathcal{O}(n^{-k})$. [/mm]

[mm] IB:$E\left[\summe\limits_{l=0}^{2k+2}\vektor{2k+2 \\ l}\left(\frac{1}{n}\sum\limits_{j=1}^{n}\cos(tX_{j})\right)^{2k+2-l}\cdot (-\mu)^{l}\right]=\mathcal{O}(n^{-(k+1)})$. [/mm]

Um den Induktionsschritt durchzuführen, habe ich zunächst die Binomialkoeffizienten umgeformt. Es gilt
[mm] \[ \vektor{2k+2 \\ l}=\vektor{2k \\ l-2}+2\vektor{2k \\ l-1}+\vektor{2k \\ l}\] [/mm]

Setzt man das nun ein und nutzt die Linearität des Erwartungswertes erhält man unter anderem folgenden Term
[mm] \[E\left[ \sum\limits_{l=0}^{2k+2}\vektor{2k \\ l-2}\left(\frac{1}{n}\sum\limits_{j=1}^{n}\cos(tX_{j})\right)^{2k+2-l}\cdot (-\mu)^{l} \right] =E\left[ \sum\limits_{l=2}^{2k+2}\vektor{2k \\ l-2}\left(\frac{1}{n}\sum\limits_{j=1}^{n}\cos(tX_{j})\right)^{2k+2-l}\cdot (-\mu)^{l} \right] \], [/mm]
da Binomialkoeffizienten  [mm] $\vektor{n \\ k}$ [/mm] für k<0 verschwinden. Jetzt kann ich eine Indexverschieben $s=l-2$ machen und erhalte
[mm] \[E\left[ \sum\limits_{s=0}^{2k}\vektor{2k \\ s}\left(\frac{1}{n}\sum\limits_{j=1}^{n}\cos(tX_{j})\right)^{2k-s}\cdot (-\mu)^{s+2} \right] =\mu^2\mathcal{O}(n^{-k})\], [/mm] nach Induktionsvoraussetzung.

Da [mm] $\mu$ [/mm] nicht von $n$ abhängt, erhalte ich eine Folge welche zu langsam konvergiert. Da sich die Konvergenzgeschwindigkeit immer nach der am langsamsten konvergierenden Folge richtet.

Edit: Ich habe wohl nur gezeigt, dass man dieses Problem nicht im Induktion lösen kann, da ja jede Folge der Ordnung [mm] $\mathcal{O}(n^{-k-1})$ [/mm] auch von der Ordnung [mm] $\mathcal{O}(n^{-k}$) [/mm] konvergiert. Für andere Ideen zur Lösung der Aufgabe wäre ich dankbar
Vielen Dank für die Mühe  

        
Bezug
Konvergenzgeschwindigeit: Mitteilung
Status: (Mitteilung) Reaktion unnötig Status 
Datum: 12:24 So 22.01.2012
Autor: blascowitz

Hallo,

das Konvergenz vorliegt habe ich nun gezeigt. Dazu habe ich den Satz von Pratt (siehe Elstrodt Maß und Integrationstheorie, S.261) verwendet.  Man kann ja
[mm] \[ E\left[ \left( \frac{1}{n}\sum\limits_{j=1}^{n}\cos(t(X_{j}))-\mu\right)^{2k} \right] \] [/mm]
als Konvergenz im $2k$-ten Mittel gegen eine entartete ZV auffassen.

Zur Konvergenzgeschwindigkeit habe ich bis jetzt nur den Satz von Berry-Essien gefunden. Allerdings würde ich so einen ähnlichen Satz  für die [mm] $L_{p}$-Konvergenz [/mm] brauchen. Gibt es sowas?

Viele Grüße
Blasco

Edit: Mir ist aufgefallen, dass  ich  auch einfach den Satz über majorisierte Konvergenz im [mm] $L_{2k}$ [/mm] hätte verwenden können um die Konvergenz zu zeigen. An Sätzen über Konvergenzraten bin ich aber nachwievor interessiert

Bezug
        
Bezug
Konvergenzgeschwindigeit: Fälligkeit abgelaufen
Status: (Mitteilung) Reaktion unnötig Status 
Datum: 14:20 Mi 25.01.2012
Autor: matux

$MATUXTEXT(ueberfaellige_frage)
Bezug
Ansicht: [ geschachtelt ] | ^ Forum "mathematische Statistik"  | ^^ Alle Foren  | ^ Forenbaum  | Materialien


^ Seitenanfang ^
ev.vorhilfe.de
[ Startseite | Mitglieder | Impressum ]